Đến nội dung

I Love MC nội dung

Có 1000 mục bởi I Love MC (Tìm giới hạn từ 05-05-2020)



Sắp theo                Sắp xếp  

#504667 Tính giá trị biểu thức $A=x^{5}+\frac{1}{x...

Đã gửi bởi I Love MC on 07-06-2014 - 10:51 trong Đại số

$x^2-4x+1=0$ nên $x^2-4x+4=3$ <--> $(x-2)^2=3$ <--> $x \in {\sqrt{3}+2;-\sqrt{3}+2}$ bây giờ thế vào tính.




#504670 Tìm tất cả các số nguyên dương a, b, c sao cho $\frac{1}...

Đã gửi bởi I Love MC on 07-06-2014 - 10:56 trong Số học

b) Gọi $A=\frac{1}{a}+\frac{1}{b}+\frac{1}{c}$ 
Ta có $A \le 1+1+1=3$ 
Do đó $A$ chỉ có thể nhận 1  trong 3 giá trị đó là $1,2,3$ bây giờ thì dễ rồi.




#504694 $\frac{1}{ab+a+2} + \frac{1}{bc+b+2} + \frac{1}{ca+c+2}...

Đã gửi bởi I Love MC on 07-06-2014 - 12:28 trong Bất đẳng thức và cực trị

$\frac{1}{a+1}+\frac{1}{ab+1}=\frac{1}{a+1}+\frac{c}{1+c} \ge \frac{4}{a+2+ab}$ tương tự suy ra điều phải c/m




#504697 Khoảng trời dành cho sự sáng tạo

Đã gửi bởi I Love MC on 07-06-2014 - 12:32 trong Bất đẳng thức và cực trị

Mình chế được cái bày này ( không biết đụng hàng ai chưa ) 
Cho $\sqrt{\frac{x}{y+z}}=a$ $\sqrt{\frac{y}{x+z}}=b$ $\sqrt{\frac{z}{x+y}}=c$ với $x,y,z>0$ 
C/m $a.b+bc+ac>2,5$




#504701 Cho x,y là 2 số dương thỏa mãn $x^{5}+y^{5}=x^...

Đã gửi bởi I Love MC on 07-06-2014 - 12:50 trong Bất đẳng thức và cực trị

Câu 2 phải là cho $a,b,c>0$ sao cho $abc=1$ 
C/m $\sum \frac{a^3}{(1+b)(c+1)} \ge \frac{3}{4}$ 
Nếu như vậy thì $\frac{a^3}{(1+b)(c+1)}+\frac{1+b}{8}+\frac{c+1}{8} \ge \frac{3a}{4}$ theo Cauchy. 
Tương tự suy ra $VT \ge $\frac{2(a+b+c)-3}{4} \ge \frac{6-3}{4}=VP$ 
Dấu bằng xảy ra khi $x=y=z=1$
 




#504702 Topic về Phương trình

Đã gửi bởi I Love MC on 07-06-2014 - 12:55 trong Phương trình, hệ phương trình và bất phương trình

1/$\frac{1}{x}+\frac{1}{x+2}+\frac{1}{x+5}+\frac{1}{x+7}=\frac{1}{x+1}+\frac{1}{x+3}+\frac{1}{x+4}+\frac{1}{x+6}$

2/ $\frac{(1995-x)^2+(1995-x)(x-1996)+(x-1996)^2}{(1995-x)^2-(1995-x)(x-1996)+(x-1996)^2}=\frac{19}{49} $

3/$5\sqrt{x^3-1}=2(x^2+2)$

4/$x^2+\sqrt{x+72}=72$

5/$(x-\sqrt{2})^3+(x+\sqrt{3})^3+(\sqrt{2}-\sqrt{3}-2x)^3=0$

6/$(a+b+x)^3-4(a^3+b^3+x^3)-12abx=0$(a,b là tham số)

7/$x^3-3abx+a^3+b^3=0($a, b là tham số)

1) $\frac{1}{x(x+1)}+\frac{1}{(x+2)(x+3)}+...+\frac{1}{(x+5)(x+6)}=\frac{1}{x}-\frac{1}{x+6}=\frac{5}{x(x+6)}$ 
--> Pt vô nghiệm




#504705 $\frac{x^{3}}{\sqrt{4-x}...

Đã gửi bởi I Love MC on 07-06-2014 - 13:06 trong Phương trình, hệ phương trình và bất phương trình

ĐK: $x<4$ 
$VT=\frac{x^3+\sqrt{4-x}x^2-4.\sqrt{4-x}=0$ 
<--> $x^3+\sqrt{4-x}x^2=4.\sqrt{4-x}$ rồi đến đây giải pt :D




#504764 $\boxed{\text{Chuyên Đề}}$ Bất đẳng thức - Cực trị

Đã gửi bởi I Love MC on 07-06-2014 - 18:57 trong Bất đẳng thức và cực trị

1 số bài khó 
177) Cho $a_1,a_2,..,a_n>0$ với $n \ge 2$. C/m 
$(a_1^3+1)(a_2^2+1)...(a_n^3+1) \ge (a_1^2a_2+1)....(a_n^2a_1+1)$ ( Cuộc thi Czech-Slovanikia-Balan 2002) 
178) Cho $x,y,z>1$ sao cho $\sum \frac{1}{x}=2$. C/m 
$\sqrt{x+z+y} \ge \sum \sqrt{x-1}$ ( Iran 1998) 
179) C/m với $a,b,c>0$ thì : 
$\sum \frac{(b+c-a)^2}{a^2+(b+c)^2} \ge \frac{3}{5}$ ( Nhật Bản 1997)




#504821 C/mR: $2(x^3-\frac{1}{x^3})>3(x^2-\fra...

Đã gửi bởi I Love MC on 07-06-2014 - 21:51 trong Bất đẳng thức và cực trị

tham khảo tại http://diendantoanho...c1sqrtb2a-le-2/




#504831 So sánh $2012^{2013}$ và $2013^{2012}$

Đã gửi bởi I Love MC on 07-06-2014 - 22:06 trong Số học

Cách c/m bđt mà bạn nói trên 
Quy nạp 
Mệnh đề đúng với r=0 
Giả sử mệnh đề đúng với $r=k$ tức là $(1+x)^k \ge 1+r.k$
Cần c/m $(1+x)^{k+1} \ge 1+r(k+1)$  
Thậy vậy $VT \ge (1+x)(1+kx)=1+kx+x+x^2k=1+(x+1)k+kx^2 \ge 1+(k+1)x$




#504833 các chuyên đề toán đại số 9

Đã gửi bởi I Love MC on 07-06-2014 - 22:08 trong Đại số

http://idoc.vn/tai-l...-toan-lop9.html trước tiên phải ôn kiến thức cho vững rồi làm nâng cao.




#504907 Tìm GTNN của biểu thức: $A=\frac{2ab+a+b+c(ab-1)}{(a...

Đã gửi bởi I Love MC on 08-06-2014 - 10:11 trong Bất đẳng thức và cực trị

$P=\frac{1}{1+c}+\frac{ab+abc-c-1}{(a+1)(b+1)(c+1)}=\frac{1}{1+c}+\left [ \frac{ab-1}{(1+a)(1+b)}+1 \right ]-1=\frac{1}{1+c}-1+\frac{a}{1+a}+\frac{b}{1+b}=\frac{a}{1+a}+\frac{b}{1+b}-\frac{c}{1+c}$

Thật vậy, ta có $P\geq \frac{5}{12}\Leftrightarrow \left ( \frac{3}{4}-\frac{c}{1+c} \right )+\left ( \frac{b}{1+b}-\frac{2}{3} \right )+(\frac{a}{1+a}-\frac{1}{2})\geq 0\Leftrightarrow \frac{3-c}{4(1+c)}+\frac{b-2}{3(b+1)}+\frac{a-1}{2(a+1)}\geq 0$

Áp dụng phép nhóm Abel, ta có BĐT tương đương với $(3-c)(\frac{1}{4(c+1)}-\frac{1}{3(b+1)})+\left [ (3-c)+(b-2) \right ]\left [ \frac{1}{3(b+1)}-\frac{1}{2(a+1)} \right ]+\left [ (3-c)+(b-2)+(a+1) \right ]\frac{1}{2(a+1)}\geq 0\Leftrightarrow \frac{(3-c)(3b-4c+1)}{12(b+1)(c+1)}+\frac{(b+1-c)(2a-3b-1)}{6(a+1)(b+1)}+\frac{a+b-c}{2(a+1)}\geq 0$




#504908 Cmr: \frac{1}{a}+\frac{1}{b...

Đã gửi bởi I Love MC on 08-06-2014 - 10:15 trong Đại số

$|x| \ge |y|$ <--> $x^2 \ge y^2$ <--> $x^2-y^2 \ge 0$ <--> $\sqrt{x^2-y^2} \ge 0$ từ đó suy ra đpcm




#504910 $x^{2}+3=5y$

Đã gửi bởi I Love MC on 08-06-2014 - 10:22 trong Đại số

VT chỉ có thể là 
$VT \equiv 7 \euiv 2$ (mod 5) và $VT \equiv 4$ (mod 5) và $VP \equiv 0$ (mod 5) nên không có $x,y$ thỏa mãn.




#504914 Tìm GTNN của $P=x+y+\frac{10}{x+y}$

Đã gửi bởi I Love MC on 08-06-2014 - 10:30 trong Bất đẳng thức và cực trị

Tham khảo tại https://www.facebook...ntrung/?fref=ts ở  bài của Hiếu A Master.




#504916 tim max

Đã gửi bởi I Love MC on 08-06-2014 - 10:33 trong Số học

$VT=\frac{1}{x^2-6x+9+8}=\frac{1}{(x-3)^2+8} \le \frac{1}{8}$ <--> $x=3$




#504931 C/m rằng $MA_1^n+(-MA_3)^n+MA_5=MA_2^n+(-MA_4)^n+MA_n^6$ với...

Đã gửi bởi I Love MC on 08-06-2014 - 12:22 trong Hình học

Cho lục giác đều $A_1A_2A_3A_4A_5A_6$ nội tiếp đường tròn $(O;R)$ , M là điểm nằm trên cung nhỏ $A_1A_6$. C/m rằng 
$MA_1^n+(-MA_3)^n+MA_5=MA_2^n+(-MA_4)^n+MA_n^6$ với $n \in {1;2;4}$ 



#504945 Cho các số dương a,b,c,d t/m a+b+c+d=1 CMR:

Đã gửi bởi I Love MC on 08-06-2014 - 14:24 trong Bất đẳng thức và cực trị

Áp dụng Cauchy-Swarchz : 
$VT \ge \frac{(1+1+2+4)^2}{a+b+c+d}=64$




#504958 CM: $\dfrac{2n}{2n + 3}..\dfrac{2...

Đã gửi bởi I Love MC on 08-06-2014 - 15:19 trong Bất đẳng thức và cực trị

Câu này anh làm trên hocmai rồi nhỉ :D up lên cho mọi người xem.

$P(n)=\dfrac{2}{3}(\prod_{x=1}^{n}\dfrac{2x}{2x+3})<\dfrac{1}{\sqrt{(x+1)^3}}$
 
Với n=1:
 
$P(1)=\dfrac{2}{3}(\prod_{x=1}^{1}\dfrac{2x}{2x+3})<\dfrac{1}{\sqrt{(1+1)^3}}$ (Đúng)
 
Giả sử BĐT đúng với $n=k$ ta sẽ chứng minh nó đúng với $n=k+1$, thật vậy:
 
Trước hết ta có:
 
$9(k+1)^3(2k+5)^2-4(k+2)^3(2k+2)^2=(k+1)^2(20k^3+120k^2+213k+97) >0 ( \forall k>0)$
 
Nên: $(\dfrac{2k+2}{2k+5})^2<\dfrac{9}{4}\dfrac{(k+1)^3}{(k+2)^3} \\\Leftrightarrow  \dfrac{2k+2}{2k+5}<\dfrac{3}{2}.\sqrt{\dfrac{(k+1)^3}{(k+2)^3}} \\\Leftrightarrow \dfrac{2}{3}.\dfrac{1}{\sqrt{(1+k)^3}}.\dfrac{2k+2}{2k+5}<\dfrac{1}{\sqrt{(k+2)^3}}$
 
Nên:
 
$P(k+1)=\dfrac{2}{3}(\prod_{x=1}^{k+1}\dfrac{2x}{2x+3})< \dfrac{2}{3}.\dfrac{1}{\sqrt{(1+k)^3}}. \dfrac{2k+2}{2k+5}<\dfrac{1}{\sqrt{(k+2)^3}}$
 
Vậy BDT đúng với $k+1$ nên theo nguyên lí qui nạp ta có đpcm.



#505051 Tìm 2 số nguyên a, b thỏa mãn $\frac{1}{a-1996}...

Đã gửi bởi I Love MC on 08-06-2014 - 19:56 trong Số học

Đặt $a-1996=m,v-2013=n$ 
$VT=\frac{1}{m}+\frac{1}{n}=1$  
Rõ ràng 1 trong $m,n$ phải có 1 số bằng 2 vì nếu $m,n>2$ thì suy ra 
$VT<1$ 
Xét $m=2$ --> $n=2$ tương tự tìm ra a,b




#505054 Hai bài BĐT và Cực trị hay

Đã gửi bởi I Love MC on 08-06-2014 - 20:14 trong Bất đẳng thức và cực trị

1) $VT \ge \dfrac{(x+y+z)^2}{a^2+b^2+c^2}=\dfrac{x^2+y^2+z^2+2(xy+yz+zx)}{a^2+b^2+c^2}>\dfrac{2(x^2+y^2+z^2)}{a^2+b^2+c^2}$ 
Trích từ diedanhocmai.com




#506141 $p(p(p(x)))$=0

Đã gửi bởi I Love MC on 12-06-2014 - 21:15 trong Đại số

Đây hình như là câu 5 IMO 2006 không biết đúng không nữa  :wacko:  :lol:




#506146 TÌm tất cả số nguyên x sao cho $(X^{3}-2X^{2}+7x-7)...

Đã gửi bởi I Love MC on 12-06-2014 - 21:25 trong Số học

Thực hiện phép chia ta được thương là $x-2$ dư là $4.x-1$ do đó $4.x-1=0 \leftrightarrow x=\frac{1}{4}$ mình không có máy tính nên không biết thực hiện đúng không :D




#506837 Chứng minh rằng phân số sau đây tối giản với mọi $n \in Z$:...

Đã gửi bởi I Love MC on 15-06-2014 - 12:04 trong Số học

Tổng của một số hữu tỉ và một số vô tỉ có thể là một số hữu tỉ không? Tổng có hai số vô tỉ có thể là một số hữu tỉ được không ?

a) Giả sử : $\frac{a}{b}+x=y$ ($y$ là số hữu tỉ,$x$ là số vô tỉ) 
$\rightarrow \frac{a}{b}-y=-x$ có $\frac{a}{b}-y$ là số hữu tỉ $-x$ vô tỉ do đó $VT # VP$ 
Kết luận ........ 
b) Tương tự.



#506975 Đề thi vào lớp 10 THPT Chuyên Hà -Tĩnh năm học 2014-2015

Đã gửi bởi I Love MC on 15-06-2014 - 20:47 trong Tài liệu - Đề thi

 

SỞ GIÁO DỤC VÀ ĐÀO TẠO                         KÌ THI TUYỂN SINH VÀO LỚP 10 TRƯỜNG THPT CHUYÊN HÀ TĨNH

       Hà Tĩnh                                                          NĂM HỌC 2014-2015

                                                                Môn thi : TOÁN ( Chung cho mọi học sinh)

                                                                                            Thời gian làm bài: 120 phút 

 

 

Câu 1. Cho $P=(\frac{-x}{\sqrt{x}(x-9)}+\frac{2}{\sqrt{x}-3}-\frac{1}{\sqrt{x}+3}): (\sqrt{x}+3 - \frac{x}{\sqrt{x}-3})$  , với $x> 0, x\neq 9$ .

              a) Rút gọn biểu thức P.

              b) Tìm giá trị của $x$ sao cho $P= \frac{-1}{4}$

Câu 2. Cho phương trình $x^{2}-2(m-2)x+m^{2}-2m+2=0$ ($m$ là tham số )

              a) Giải phương trình khi $m=-1$

              b) Tìm tất cả các giá trị của $m$ để phương trình có 2 nghiệm phân biệt $x_{1},x_{2}$ thỏa mãn: $\left | 2(x_{1}+x_{2})+x_{1}x_{2} \right |=3$ .

Câu 3.  a) Giải phương trình  $\sqrt{2x+3}-2\sqrt{x+1}=-1$

             b) Giải hệ phương trình $\left\{\begin{matrix} xy^{2}+2y^{2}-2=x^{2}+3x & & \\ x+y=3\sqrt{y-1} & & \end{matrix}\right.$

Câu 4.  Cho tam giác nhọn $ABC$ nội tiếp đường tròn tâm $O$ , có $\widehat{BAC}=45^{\circ}$ , $BC=a$ . 

             Gọi $E,F$ tương ứng là chân đường vuông góc hạ từ $B$ xuống $AC$ , từ $C$ xuống $AB$ . Gọi $I$ là điểm đối xứng của $O$ qua $EF$ .

               a) Chứng minh $BFOC,AEIF$ là các tứ giác nội tiếp đường tròn.

               b) Tính $EF$ theo $a$ .

Câu 5. Biết phương trình $x^{4}+ax^{3}+bx^{2}+ax+1=0$ có nghiệm .

            Chứng minh $a^{2}+b^{2}\geqslant \frac{4}{5}$ .

    -HẾT-

 

Thí sinh không sử dụng tài liệu 

Giám thị không giải thích gì thêm .

Hết 

 

 Cho phương trình $x^{2}-2(m-2)x+m^{2}-2m+2=0$ ($m$ là tham số )

              a) Giải phương trình khi $m=-1$ 
Trình còn yếu :D 
$x^2+6x+1+2+2=0 \leftrightarrow (x+3)^2=4 \leftrightarrow x \in {-5;-1}$